Stuck at proving a bounded above Subsequence

In summary: But I don't know how else to justify the choice of ## n_{k+1} ## and ## n_{k+2} ## and so on.You can justify the choice of ##n_{k+1}##, ##n_{k+2}##, etc. by simply stating that since we know there exists an ##n>n_k## such that ##a_n<L##, we can choose that ##n## to be the next element in our subsequence. This follows the definition of a subsequence, which is a sequence of elements taken from the original sequence in the order they appear.
  • #1
CGandC
326
34
Summary:: x

Let ## \{ a_{n} \} ## be a sequence.
Prove: If for all ## N \in { \bf{N} } ## there exists ## n> N ## such that ## a_{n} \leq L ## , then there exists a subsequence ## \{ a_{n_{k}} \} ## such that ##
a_{n_{k}} \leq L ##

My attempt:
Suppose that for all ## N \in {\bf{N}} ## there exists ## n> N ##, such that ## a_{n} \leq L ##. Since ## \{ a_{n} \} ## is a sequence, there exists a subsequence ## \{ a_{n_{k}} \}## such that there exists a monotonically increasing sequence of natural numbers ## \{ n_{k} \} ## such that ## \forall k \in {\bf{N}}, n_{k} \geq k ## . Let ## k \in {\bf{N}} ## be arbitrary so there exists ## n_{k} \geq k ##. Thus ( by universal instantiation of ## n_{k} ## in the statement " for all ## N \in {\bf{N}} ## there ... " ) there exists ## n > n_{k} > k ## such that ## a_{n} \leq L ##

However ,I don't know if ## a_{n_{k}} ## is less than or greater than ## a_{n} ## . So I'm stuck, how am I supposed to proceed from here to show that ## a_{n_{k}} \leq L ## ?
 
Physics news on Phys.org
  • #2
CGandC said:
Thus ( by universal instantiation of ## n_{k} ## in the statement " for all ## N \in {\bf{N}} ## there ... " ) there exists ## n > n_{k} > k ## such that ## a_{n} \leq L ##
That is correct but you are not finding the right sequence here.
However ,I don't know if ## a_{n_{k}} ## is less than or greater than ## a_{n} ##
Why would that matter?
So I'm stuck, how am I supposed to proceed from here to show that ## a_{n_{k}} \leq L ## ?
You can't because it won't be true for any n_k you come up with. You need to define n_k based on the sequence a. The best approach is to do this one element at a time. Let's say a_1, a_6, a_245 are three elements smaller than L. Can you find a fourth one? Once you have that, can you find a fifth one? Generalize and you get a full sequence.
 
  • #3
Maybe the formal generalization goes as follows :

Let there be arbitrary ## N' \in { \bf{N} } ##, so there exists ## n > N' ## such that ## a_{n} \leq L ##.
We'll choose ## k' \in { \bf{N} } ## so that there exists ## n_{k'} \in { \bf{N} } ## ( " there exists ## n_{k'} \in { \bf{N} } ## " because since ## k' ## is a specific value and not arbitrary, then ## n_{k'} ## is also a specific value and not arbitrary ) such that ## n_{k'} = n ##. So it immediately follows that ## a_{n} = a_{n_{k'}} \leq L ##

Is this right?
 
  • #4
You spend a lot of , big words justifying the existence of ##n_{k'}## that both doesn't seem necessary to me and sounds very confusing.I would just say something like, given ##n_1,...,n_k## in the subsequence, we know there is some ##n>n_k## such that ##a_n<L##. Let ##n_{k+1}=n##. And do this repeatedly to form the sequence.
 
  • Like
Likes CGandC, Delta2 and mfb
  • #5
So I can say the following?:
We know there is some ## n>n_k ## such that ## a_n < L ## . Let ## n_{k+1} = n ## .
We know there is some ## n>n_{k+1} ## such that ## a_n < L ## . Let ## n_{k+2} = n ## .
We know there is some ## n>n_{k+2} ## such that ## a_n < L ## . Let ## n_{k+3} = n ## .
.
.
.
...

So that the new subsequence we form is ## ( a_{n_k+1} , a_{n_{k+2}} , a_{n_{k+3}} , ... ) ## ?
 
  • Like
Likes mfb
  • #6
The point of what I wrote is that I assumed we already had the first k elements of the sequence which you dropped from your final list, but your sequence also works. I would consider what you wrote a proof.

It's possible if you're taking a class they will require you to write something more formal, but

1.) Most of the mathematical community doesn't actually speak this way talking about universal instantiation etc.
2.) I think it's mostly a blocker to understanding the important point.

If you think what you wrote is not acceptable for a person grading this and you're not sure how to turn it into something acceptable we can discuss it.
 
  • #7
I think my difficulty comes from trying to apply the definition of a subsequence ( from the book Introduction to Real Analysis . Robert G. Bartle , Donald R. Sherbert ):

1606722297372.png


Does this definition tell me:
1. If I have a sequence ## ( x_n ) ## and an increasing sequence of natural numbers ## ( n_k ) ## , then there exists a new sequence ## X' = ( x_{n_{k}} ) = ( x_{n_{1}} , x_{n_{2}}, ... , x_{n_{k}} , ... ) ##

Or

2. for all sequence ## ( x_n ) ## and for all increasing sequence of natural numbers ## ( n_k ) ## , then there exists a new sequence ## X' = ( x_{n_{k}} ) = ( x_{n_{1}} , x_{n_{2}}, ... , x_{n_{k}} , ... ) ##

Or

3. for all sequence ## ( x_n ) ## ,there exists an increasing sequence of natural numbers ## ( n_k ) ## , then there exists a new sequence ## X' = ( x_{n_{k}} ) = ( x_{n_{1}} , x_{n_{2}}, ... , x_{n_{k}} , ... ) ##

Which is correct ,and basically what is the logical format of the above definition?
I'm used to being able to use definitions and proven statements which start with quantifiers like " there exists " and " for all ". But in the above definition I have " Let ", so I feel like I don't exactly know how to use the definition ( not the first time I encountered such definition ).
 

Attachments

  • 1606719853289.png
    1606719853289.png
    14.1 KB · Views: 117
  • #8
1 and 2 are both correct.
X' depends on the sequence nk of course.

You are overthinking this.

Here is a sequence:
##x_1, x_2, x_3, x_4, x_5, x_6, ...##
Here is a subsequence I chose:
##x_2, x_4, x_8, x_{16}, ...##, here nk = 2,4,8,16,...
Here is another subsequence:
##x_1, x_3, x_5, x_7, ...##, here nk = 1,3,5,7,...

nk is just a list telling you which elements to use from the original sequence.
 
  • Like
Likes CGandC
  • #9
Basically your problem in the OP is to give a proper and possibly recursive definition of the sequence ##n_k## and then prove that ##n_k## is strictly increasing and also prove that ##a_{n_k}\leq L##. If you define properly the sequence ##n_k## then those two proofs will be immediate consequence of the proper definition.
 
Last edited:

What does it mean for a sequence to be bounded above?

A sequence is considered to be bounded above if there exists a number, known as an upper bound, such that all terms in the sequence are less than or equal to this number. In other words, the sequence does not grow infinitely large and is limited by a specific value.

How do you prove that a sequence is bounded above?

To prove that a sequence is bounded above, you must show that there exists an upper bound that satisfies the definition mentioned above. This can be done by finding a specific number or expression that is greater than or equal to all terms in the sequence.

What is a bounded above subsequence?

A bounded above subsequence is a subset of a sequence that is itself bounded above. This means that the terms in the subsequence are all less than or equal to a specific number, which is also an upper bound for the entire sequence.

How do you prove the existence of a bounded above subsequence?

To prove the existence of a bounded above subsequence, you must first show that the original sequence is bounded above. Then, you can use a method such as the Bolzano-Weierstrass theorem to show that there exists a convergent subsequence within the original sequence. This subsequence will also be bounded above, as it is a subset of the original bounded above sequence.

Why is proving the existence of a bounded above subsequence important?

Proving the existence of a bounded above subsequence is important in many areas of mathematics, particularly in the study of sequences and series. It allows us to show that a sequence has a limit or a convergent subsequence, which can help us understand the behavior of the sequence as a whole. Additionally, it can be used in more complex proofs and theorems to establish the existence of certain properties or solutions.

Similar threads

  • Math Proof Training and Practice
Replies
6
Views
1K
  • Math Proof Training and Practice
Replies
10
Views
1K
  • Math Proof Training and Practice
Replies
8
Views
1K
Replies
7
Views
2K
Replies
2
Views
136
  • Math Proof Training and Practice
Replies
4
Views
1K
  • Calculus and Beyond Homework Help
Replies
4
Views
1K
  • Math Proof Training and Practice
Replies
5
Views
886
  • Calculus and Beyond Homework Help
Replies
11
Views
1K
  • Math Proof Training and Practice
3
Replies
100
Views
7K
Back
Top